Difference between revisions of "2023 AMC 12A Problems/Problem 9"

(Created page with "do little worthless cheater go grind amc if you wanna suceed best of luck")
 
(redirect)
Line 1: Line 1:
do little worthless cheater
+
==Problem==
go grind amc if you wanna suceed
+
A square of area <math>2</math> is inscribed in a square of area <math>3</math>, creating four congruent triangles, as shown below. What is the ratio of the shorter leg to the longer leg in the shaded right triangle?
best of luck
+
<asy>
 +
size(200);
 +
defaultpen(linewidth(0.6pt)+fontsize(10pt));
 +
real y = sqrt(3);
 +
pair A,B,C,D,E,F,G,H;
 +
A = (0,0);
 +
B = (0,y);
 +
C = (y,y);
 +
D = (y,0);
 +
E = ((y + 1)/2,y);
 +
F = (y, (y - 1)/2);
 +
G = ((y - 1)/2, 0);
 +
H = (0,(y + 1)/2);
 +
fill(H--B--E--cycle, gray);
 +
draw(A--B--C--D--cycle);
 +
draw(E--F--G--H--cycle);
 +
</asy>
 +
 
 +
<math>\textbf{(A) }\frac15\qquad\textbf{(B) }\frac14\qquad\textbf{(C) }2-\sqrt3\qquad\textbf{(D) }\sqrt3-\sqrt2\qquad\textbf{(E) }\sqrt2-1</math>
 +
 
 +
==Solution==
 +
 
 +
Note that each side length is <math>\sqrt{2}</math> and <math>\sqrt{3}.</math> Let the shorter side of our triangle be <math>x</math>, thus the longer leg is <math>\sqrt{3}-x</math>. Hence, by the Pythagorean Theorem, we have <cmath>(x-\sqrt{3})^2+x^2=2</cmath>
 +
<cmath>2x^2-2x\sqrt{3}+1=0</cmath>.
 +
 
 +
By the quadratic formula, we find <math>x=\frac{\sqrt{3}\pm1}{2}</math>. Hence, our answer is <math>\frac{\sqrt{3}-1}{\sqrt{3}+1}=\boxed{\textbf{(C) }2-\sqrt3}.</math>
 +
 
 +
~SirAppel ~ItsMeNoobieboy
 +
 
 +
==Solution 2 (Area Variation of Solution 1)==
 +
 
 +
Looking at the diagram, knowing the square inscribed the square with area 3 is area 2. We would automatically know the area of the triangles are <math>\frac{1}{4}</math>
 +
 
 +
From solution 1, the base is <math>x</math> and the height <math>\sqrt{3} - x</math>. Which means <math>\frac{x(\sqrt{3} - x)}{2} = \frac{1}{4}</math>
 +
 
 +
We can turn this into a quadratic equation making it <math>x^2-x\sqrt{3}+\frac{1}{2} = 0</math>
 +
 
 +
By using the quadratic formula, we get <math>x=\frac{\sqrt{3}\pm1}{2}</math>.Therefore, the answer is <math>\frac{\sqrt{3}-1}{\sqrt{3}+1}=\boxed{\textbf{(C) }2-\sqrt3}.</math>
 +
 
 +
~ghfhgvghj10 (If I made any mistakes, feel free to make minor edits)
 +
 
 +
==See Also==
 +
{{AMC10 box|year=2023|ab=A|num-b=10|num-a=12}}
 +
{{AMC12 box|year=2023|ab=A|num-b=8|num-a=10}}
 +
{{MAA Notice}}

Revision as of 23:44, 9 November 2023

Problem

A square of area $2$ is inscribed in a square of area $3$, creating four congruent triangles, as shown below. What is the ratio of the shorter leg to the longer leg in the shaded right triangle? [asy] size(200); defaultpen(linewidth(0.6pt)+fontsize(10pt)); real y = sqrt(3); pair A,B,C,D,E,F,G,H; A = (0,0); B = (0,y); C = (y,y); D = (y,0); E = ((y + 1)/2,y); F = (y, (y - 1)/2); G = ((y - 1)/2, 0); H = (0,(y + 1)/2); fill(H--B--E--cycle, gray); draw(A--B--C--D--cycle); draw(E--F--G--H--cycle); [/asy]

$\textbf{(A) }\frac15\qquad\textbf{(B) }\frac14\qquad\textbf{(C) }2-\sqrt3\qquad\textbf{(D) }\sqrt3-\sqrt2\qquad\textbf{(E) }\sqrt2-1$

Solution

Note that each side length is $\sqrt{2}$ and $\sqrt{3}.$ Let the shorter side of our triangle be $x$, thus the longer leg is $\sqrt{3}-x$. Hence, by the Pythagorean Theorem, we have \[(x-\sqrt{3})^2+x^2=2\] \[2x^2-2x\sqrt{3}+1=0\].

By the quadratic formula, we find $x=\frac{\sqrt{3}\pm1}{2}$. Hence, our answer is $\frac{\sqrt{3}-1}{\sqrt{3}+1}=\boxed{\textbf{(C) }2-\sqrt3}.$

~SirAppel ~ItsMeNoobieboy

Solution 2 (Area Variation of Solution 1)

Looking at the diagram, knowing the square inscribed the square with area 3 is area 2. We would automatically know the area of the triangles are $\frac{1}{4}$

From solution 1, the base is $x$ and the height $\sqrt{3} - x$. Which means $\frac{x(\sqrt{3} - x)}{2} = \frac{1}{4}$

We can turn this into a quadratic equation making it $x^2-x\sqrt{3}+\frac{1}{2} = 0$

By using the quadratic formula, we get $x=\frac{\sqrt{3}\pm1}{2}$.Therefore, the answer is $\frac{\sqrt{3}-1}{\sqrt{3}+1}=\boxed{\textbf{(C) }2-\sqrt3}.$

~ghfhgvghj10 (If I made any mistakes, feel free to make minor edits)

See Also

2023 AMC 10A (ProblemsAnswer KeyResources)
Preceded by
Problem 10
Followed by
Problem 12
1 2 3 4 5 6 7 8 9 10 11 12 13 14 15 16 17 18 19 20 21 22 23 24 25
All AMC 10 Problems and Solutions
2023 AMC 12A (ProblemsAnswer KeyResources)
Preceded by
Problem 8
Followed by
Problem 10
1 2 3 4 5 6 7 8 9 10 11 12 13 14 15 16 17 18 19 20 21 22 23 24 25
All AMC 12 Problems and Solutions

The problems on this page are copyrighted by the Mathematical Association of America's American Mathematics Competitions. AMC logo.png